Hardware Upgrade Forum

Hardware Upgrade Forum (https://www.hwupgrade.it/forum/index.php)
-   Scienza e tecnica (https://www.hwupgrade.it/forum/forumdisplay.php?f=91)
-   -   [Official Thread]Richieste d'aiuto in MATEMATICA: postate qui! (https://www.hwupgrade.it/forum/showthread.php?t=1221191)


Demin Black Off 11-09-2007 22:42

Ho un problemone abbastanza complesso :cry: complesso per me che non metto mano a geo da 5 anni...

Supponiamo di avere una sfera in 3d ( ogni punto è dal tipo (x,y,z) ).

Supponiamo che la sfera è pogiata sul piano XY ( Z è la quota ).

Supponiamo che la sfera sia vuota e che tra sfera è piano ci sia attrito in modo tale che la sfera se si muove ruota ( non creiamo altri problemi :P ). Quindi consideriamo solo Traslazione E Rotazione insieme.

Il problema è, dato uno spostamentro arbitrario (x1,y1,0) calcolare angolo di rotazione e direzione di rotazione.

MaxArt 12-09-2007 03:00

La rotazione come la vuoi espressa? Suppongo come l'angolo formato con il sistema di riferimento cartesiano.
Allora l'angolo lo ottieni con l'arcotangente di y1/x1 (o l'arcocotangente di x1/y1), e aggiusti la direzionea seconda dei segni (dipende dalla convenzione: solitamente, se x1<0 allora aggiungi un angolo piatto).
Per l'angolo di rotazione, calcoli la distanza di (x1, y1, 0) dall'origine e dividi per il raggio della sfera: ottieni l'angolo in radianti.
Cosa c'entra il fatto che la sfera sia vuota? :confused: E non avresti potuto dire semplicemente che "ruota senza strisciare"?

Fenomeno85 12-09-2007 08:57

Quote:

Originariamente inviato da Ziosilvio (Messaggio 18620657)
La matrice rappresentativa dell'applicazione lineare F rispetto alle basi U e W, è la matrice A tale che a{i,j} è la coordinata rispetto a w{j} dell'immagine di b{i} mediante f.

grazie per la risposta, ma b chi è?

~§~ Sempre E Solo Lei ~§~

Demin Black Off 12-09-2007 09:19

Quote:

Originariamente inviato da MaxArt (Messaggio 18655863)
La rotazione come la vuoi espressa? Suppongo come l'angolo formato con il sistema di riferimento cartesiano.
Allora l'angolo lo ottieni con l'arcotangente di y1/x1 (o l'arcocotangente di x1/y1), e aggiusti la direzionea seconda dei segni (dipende dalla convenzione: solitamente, se x1<0 allora aggiungi un angolo piatto).
Per l'angolo di rotazione, calcoli la distanza di (x1, y1, 0) dall'origine e dividi per il raggio della sfera: ottieni l'angolo in radianti.
Cosa c'entra il fatto che la sfera sia vuota? :confused: E non avresti potuto dire semplicemente che "ruota senza strisciare"?

Ho scritto che era vuota perchè il problema dopo dice :

Dato ogni punto (x,y,z) della sfera, calcolare dove si trova il nuovo punto :D

Ora inizio a fare questa prima parte, grazie ;) Questa seconda parte penso sia riconducibile al prodotto della matrice di punti (x,y,z) per una matrice di rotazione da come ho capito e poi applico la traslazione.

Ziosilvio 12-09-2007 09:51

Quote:

Originariamente inviato da Fenomeno85 (Messaggio 18657149)
grazie per la risposta, ma b chi è?

È l'i-esimo elemento di U.

Ragion per cui dovrebbe chiamarsi u{i}. Ora edito il post originale.

Demin Black Off 12-09-2007 10:32

Quote:

Originariamente inviato da Demin Black Off (Messaggio 18657471)
Ho scritto che era vuota perchè il problema dopo dice :

Dato ogni punto (x,y,z) della sfera, calcolare dove si trova il nuovo punto :D

Ora inizio a fare questa prima parte, grazie ;) Questa seconda parte penso sia riconducibile al prodotto della matrice di punti (x,y,z) per una matrice di rotazione da come ho capito e poi applico la traslazione.

Allora, dato l'angolo di rotazione ho applicato la matrice di rotazione lungo x e lungo y.

Il problema è che non mi trovo bene la rotazione lungo y, è come se l'asse fosse sballato quando faccio la seconda rotazione, come è possibile ?!

viceversa se applico prima quella lungo y e poi quella lungo x, è quella lungo x ad essere quasi sempre sballata :(

MaxArt 12-09-2007 10:53

Quote:

Originariamente inviato da Demin Black Off (Messaggio 18658752)
Allora, dato l'angolo di rotazione ho applicato la matrice di rotazione lungo x e lungo y.

Il problema è che non mi trovo bene la rotazione lungo y, è come se l'asse fosse sballato quando faccio la seconda rotazione, come è possibile ?!

viceversa se applico prima quella lungo y e poi quella lungo x, è quella lungo x ad essere quasi sempre sballata :(

Non ho capito cos'hai fatto (o meglio, ho una qualche idea ma non capisco perché l'avresti dovuta usare). Puoi spiegarmelo nel dettaglio?
Come la vuoi far muovere la sfera? Prima lungo l'asse x e poi lungo l'asse y? O sulla retta che la congiunge con (x1, y1, 0)?
Cosa vuoi ottenere alla fine?

Thunderx 12-09-2007 11:19

non è che potrei postare un esercizio di teoria dei fenomeni aleatori su cui sono totalmente in panne:D :D ???

Demin Black Off 12-09-2007 12:17

Quote:

Originariamente inviato da MaxArt (Messaggio 18659134)
Non ho capito cos'hai fatto (o meglio, ho una qualche idea ma non capisco perché l'avresti dovuta usare). Puoi spiegarmelo nel dettaglio?
Come la vuoi far muovere la sfera? Prima lungo l'asse x e poi lungo l'asse y? O sulla retta che la congiunge con (x1, y1, 0)?
Cosa vuoi ottenere alla fine?

Ma non è la stessa cosa far muovere prima lungo x e poi lunho y, o lungo la retta che cogiugente ???

A livello pratico, devo far rotolare correttamente questa palla :)



Riesco a farla rotolare correttamente solo lungo SOLO x o SOLO z.

Io applico prima la rotazione lungo x e poi quella lungo z, penso sia sbagliato :(

Lucrezio 12-09-2007 12:22

Quote:

Originariamente inviato da Demin Black Off (Messaggio 18660688)
Io applico prima la rotazione lungo x e poi quella lungo z, penso sia sbagliato :(

Occhio!
Le rotazioni non infinitesime non commutano :D
Forse il problema è quello.... ;)

MaxArt 12-09-2007 15:19

Quote:

Originariamente inviato da Demin Black Off (Messaggio 18660688)
Ma non è la stessa cosa far muovere prima lungo x e poi lunho y, o lungo la retta che cogiugente ???

Dipende! Se è una traslazione sì, se è una rotazione no. Per rendertene conto, prendi un libro: prima lo ruoti di 90° lungo un asse e poi di altri 90° lungo un altro. Poi lo rimetti nella posizione iniziale e fai gli stessi movimenti ma prima effettui il secondo e viceversa. Il libro non si troverà nella stessa posizione ;)
E' come ha detto Lucrezio. In termini matematici, le rotazioni nello spazio tridimensionale non sono un gruppo commutativo. ;)

Quote:

A livello pratico, devo far rotolare correttamente questa palla :)
Ok, ma cosa intendi con rotolare "correttamente"? E' per questo che ti chiedevo se dovessi muoverla per forza lungo gli assi.
Comunque, il piano non era XY? :confused: Com'è che ora è XZ?

Demin Black Off 12-09-2007 18:24

Ho capito i "problemi".

Il primo errore che commettevo è appunto credere che le due rotazioni fossero communtative.

Il secondo è che facevo sempre prima la rotazione della palla nell'origine lungo x e poi lungo z e poi applicavo la traslazione... Questo faceva sballare tutto se la palla aveva fatto un determinato percorso la rotazione non diventava "reale".

Ora è tutto OK !! dato che me la memorizzo alla base la rotazione e di volta in volta faccio la traslazione :)

stgww 13-09-2007 14:53

Scusate che significa quando nella dispersione quadratica media mi dice di indicare x come iesima misura di una serie di misure ?
Thx

Ziosilvio 14-09-2007 16:08

Quote:

Originariamente inviato da Antonio23 (Messaggio 18698079)
se una funzione è di quadrato sommabile in un intervallo chiuso (diciamo [-a;a]) allora essa è anche sommabile in tale intervallo?

Beh, se l'intervallo oltre che chiuso è anche limitato (anche IR è chiuso, ma L2(IR) non è contenuto in L1(IR)), allora sì.
Anzi, in realtà non serve neanche che sia chiuso, e nemmeno che sia un intervallo: basta che abbia misura di Lebesgue finita.
Quote:

posso desumerlo da questa relazione:

integrale in [-a,a] di |f(x)| è minore uguale della radice quadrata dell'integrale in [-a;a] di |f(X)|^2? (ve lo chiedo perchè non ricordo se quest'ultima relazione vale...)
Non puoi, perché la disuguaglianza è vera in generale solo se il secondo membro viene moltiplicato per la radice quadrata di 2a.
Questo segue dal ragionamento generale che si fa per dimostrare che, se A è di misura finita e f è in Lp(A) per qualche p>=1, allora f è anche in Lq(A) per ogni q compreso tra 1 e p.
Quote:

perchè una funzione è sommabile (L-integrabile) se e solo se è sommabile il suo modulo?
Per definizione.

serbring 14-09-2007 16:17

non riesco a capire una cosa sulle'equazioni differenziali alle derivate parziali,
andate nel paragrafo che illustra le equazioni quasi lineari non omogenee (PAG 12) di questo documento

http://cdm.unimo.it/home/matematica/...anuele/pde.pdf

lui ha du/ds +au=c. Come fà ad ottenere la formula successiva?

inoltre andate a PAG 27

lui ha FI''/FI=fi''/fi

con fi=fi maiuscolo FI=fi minuscolo

come fà a ricavare fi e FI?

Ziosilvio 17-09-2007 22:00

Quote:

Originariamente inviato da marcio3000 (Messaggio 18741820)
per trovare l'inverso di y = radice^3(x) + 1
devo esplicitare la x
radice^3(x) = y - 1

Giusto.
Quote:

adesso posso elevare alla terza tranquillamente?
Sì.
Quote:

A cosa devo stare attento?
A niente, la radice cubica di un numero reale è sempre definita univocamente.

serbring 18-09-2007 10:30

Quote:

Originariamente inviato da serbring (Messaggio 18699513)
non riesco a capire una cosa sulle'equazioni differenziali alle derivate parziali,
andate nel paragrafo che illustra le equazioni quasi lineari non omogenee (PAG 12) di questo documento

http://cdm.unimo.it/home/matematica/...anuele/pde.pdf

lui ha du/ds +au=c. Come fà ad ottenere la formula successiva?

inoltre andate a PAG 27

lui ha FI''/FI=fi''/fi

con fi=fi maiuscolo FI=fi minuscolo

come fà a ricavare fi e FI?

Nessuno sà aiutarmi? Scusate se sono assillante ma ho un esame venerdì...e sono un po' teso...:(

Ziosilvio 18-09-2007 11:47

Quote:

Originariamente inviato da serbring (Messaggio 18699513)
andate nel paragrafo che illustra le equazioni quasi lineari non omogenee (PAG 12) di questo documento

http://cdm.unimo.it/home/matematica/...anuele/pde.pdf

lui ha du/ds +au=c. Come fà ad ottenere la formula successiva?

Spero di non sbagliare, ma mi sembra la soluzione classica di un'equazione lineare del primo ordine in dimensione uno... caso al quale ci si sta riconducendo, perché si stanno facendo variare x, y, e u in funzione dell'ascissa curvilinea s.
Quote:

inoltre andate a PAG 27

lui ha FI''/FI=fi''/fi

con fi=fi maiuscolo FI=fi minuscolo

come fà a ricavare fi e FI?
Il sistema



si può riscrivere come un sistema di due equazioni differenziali ordinarie del secondo ordine:



La procedura per il calcolo delle soluzioni di tali equazioni è nota dai corsi di Analisi.
Imponendo le condizioni iniziali Phi(0)=phi(L)=0 e applicando la relazione



si osserva che si può cercare una soluzione come combinazione lineare di seni e coseni. Questo è esattamente ciò che viene fatto.

ROFCLAN_LOW 18-09-2007 12:10

Integrali: Problema con un Integrale Improprio! Help pls!
 
1 Allegato(i)
Spero possiate aiutarmi, non riesco in nessun modo a risolvere un integrale in realtà una serie di integrali tutti della stessa specie, il seguente:

INT[ (arctan(x-1)) / (x^a (x^2 -1)^2) dx ] da0 a +inf
http://img259.imageshack.us/my.php?i...tegralegm2.png
Ho allegato l'immagine piu chiara..:)

Vorrei capire il metodo da utilizzare per risolverlo, ho provato per parti ma si complica, penso debba essere risolto per sostituzione ma non riesco a vedere la soluzione in nessun modo.

x^a : l'esercizio chiede per quali valori di a l'integrale converge la risposta è mai, cioè non converge ma per capirlo devo comunque trovare la primitiva giusto? e come?

Grazie mille!!
Ciao.

ROFCLAN_LOW 18-09-2007 12:19

Come risolvo questo Integrale?? Aiuto!
 
1 Allegato(i)
Non riesco a risolvere questo integrale


http://img259.imageshack.us/my.php?i...tegralegm2.png

come posso fare? vorrei capire il procedimento utilizzato per la soluzione..
Grazie,

serbring 19-09-2007 11:35

Quote:

Originariamente inviato da Ziosilvio (Messaggio 18753333)
Spero di non sbagliare, ma mi sembra la soluzione classica di un'equazione lineare del primo ordine in dimensione uno... caso al quale ci si sta riconducendo, perché si stanno facendo variare x, y, e u in funzione dell'ascissa curvilinea s.

Il sistema



si può riscrivere come un sistema di due equazioni differenziali ordinarie del secondo ordine:



La procedura per il calcolo delle soluzioni di tali equazioni è nota dai corsi di Analisi.
Imponendo le condizioni iniziali Phi(0)=phi(L)=0 e applicando la relazione



si osserva che si può cercare una soluzione come combinazione lineare di seni e coseni. Questo è esattamente ciò che viene fatto.

ah grazie mille...infatti il metodo era spiegato meglio diverse pagine dopo. PAG 31 di http://cdm.unimo.it/home/matematica/...anuele/pde.pdf però non'ho capito una cosa quando risolvo il sistema di due equazione differenziali ottengo le due soluzioni generali

fi=Ce^(-kt)
FI=C1cos(radq(k)x)+C2*sen(radq(k)x)

se unisco le seguenti condizioni iniziali

u(0,t)=0 e u(1,t)=0

FI(0)=0 implica c1=0
FI(1)=0 implica u(1,t)=C2*sen(radq(k))=0 quindi radq(k)=n*pigreco

ora come faccio a calcolare C2 e C? Negli appunti lui sceglie C2=C=1. Ma non'ho capito in base a quale criterio...

BauAndrea 20-09-2007 00:45

buona sera.
ho un problema con una disequazione che può sembrare molto semplice, ma per chi si avvicina ai formalismi universitari purtroppo non lo è.

utilizzo una gif perché risparmio veramente parecchio tempo.

le soluzioni del quesito danno F.V.V. ma mi chiedo come possa essere possibile contemporaneamente la veridicità di punto due e tre. secondo i miei calcoli la soluzione esatta dovrebbe essere la 2 in quanto siamo in presenza di una disequazione p(x)>=0 con a>0. tral'altro credo che la scrittura della soluzione sia errata, visto che nelle prime pagine del libro viene detto che quando si esplica l'insieme e c'è il simbolo di infinito si usa la tonda e non la quadra.
aiutini?

tral'altro il discriminante vede k^2-4k-8>=0 (in quanto essendo sotto radice il radicando deve essere >=0 per non portarci nella situazione di avere un discriminante nullo)

Ziosilvio 20-09-2007 10:22

Quote:

Originariamente inviato da BauAndrea (Messaggio 18783678)

Cos'è I?
(Immagino sia l'insieme in cui il polinomio è positivo, ma chiedo conferma.)

poffarbacco 20-09-2007 10:32

Salve, mi aiutereste a risolvere i seguenti quesiti?
Ho la funzione: definita in [0,+inf]

(a è il parametro alfa)

Mi viene chiesto di estendere la funzione nel punto x=0 definendo la f(0)=0, e definire se è derivabile e continua. Definire poi per quali valori la funzione è iniettiva.

Grazie!

BauAndrea 20-09-2007 11:33

Quote:

Originariamente inviato da Ziosilvio (Messaggio 18786485)
Cos'è I?
(Immagino sia l'insieme in cui il polinomio è positivo, ma chiedo conferma.)

I è l'insieme delle soluzioni

Ziosilvio 20-09-2007 11:39

Quote:

Originariamente inviato da BauAndrea (Messaggio 18787835)
I è l'insieme delle soluzioni

Soluzioni di cosa?
Di un problema in cui l'incognita è k, o di un problema in cui l'incognita è x?
E poi: il problema consiste nel trovare i valori per cui il polinomio si annulla, o quelli per cui il polinomio è positivo?

BauAndrea 20-09-2007 12:04

hai ragione scusa, ci stiamo riferendo all'insieme delle soluzioni di x per cui l'equazione si annulla. (credo :( )

"Risolvere un equazione significa determinare l'insieme delle sue soluzioni. Per insieme delle soluzioni di un equazione intendiamo l'insieme I (sottoinsiemecoincidente c) CE formato dai valori per cui l'equazione è soddisfatta."

Ziosilvio 20-09-2007 12:16

Quote:

Originariamente inviato da BauAndrea (Messaggio 18788445)
hai ragione scusa, ci stiamo riferendo all'insieme delle soluzioni di x per cui l'equazione si annulla. (credo :( )

"Risolvere un equazione significa determinare l'insieme delle sue soluzioni. Per insieme delle soluzioni di un equazione intendiamo l'insieme I (sottoinsiemecoincidente c) CE formato dai valori per cui l'equazione è soddisfatta."

Grazie.
Letto meglio: si trattava di una disequazione, quindi la definizione di I diventa sensata.

Ora, quale che sia k, il coefficiente direttore del polinomio di secondo grado è sempre 1, che è positivo; quindi, quale che sia k, il polinomio di secondo grado assume segno negativo per x interno all'intervallo delimitato dalle soluzioni, e positivo all'esterno.
Quindi la 1 è falsa.

D'altra parte, il discriminante del polinomio p(x) è k^2-4(k+2)=k^2-4k+8.
Questo è un polinomio di secondo grado in k, che si annulla per k=2*(1-sqrt(3)) e k=2*(1+sqrt(3)).
Per k esterno all'intervallo delimitato da questi due valori, il discriminante del polinomio p(x) è positivo e il polinomio ha il segno del suo coefficiente direttore solo all'esterno dell'intervallo aperto delimitato dalle proprie radici reali. Quindi la 2 è vera.
Per k interno all'intervallo delimitato da questi due valori, il discriminante del polinomio p(x) è negativo e il polinomio ha sempre il segno del suo coefficiente direttore. Quindi la 3 è vera.

pietro84 20-09-2007 12:16

sistemi formali e teorema di incompletezza
 
qualcuno sa dove posso trovare del materiale sui sistemi formali e sul teorema di incompletezza?

Ziosilvio 20-09-2007 12:21

Quote:

Originariamente inviato da poffarbacco (Messaggio 18786665)
Ho la funzione: definita in [0,+inf]

(a è il parametro alfa)

Mi viene chiesto di estendere la funzione nel punto x=0 definendo la f(0)=0, e definire se è derivabile e continua.

Essendoci di mezzo un logaritmo di x, la funzione è definita solo per x>0.
Se vuoi estenderla per continuità nel modo proposto, devi dimostrare che f(x) converge a 0 per x che tende a 0 da destra; e questo dovrebbe essere un classico.

Per quanto riguarda la derivabilità, ti ricordo che la derivata è definita solo nei punti interni.
Tutt'al più, potresti calcolare la derivata destra, ossia (se esiste)


Quote:

Definire poi per quali valori la funzione è iniettiva.
Studia il segno della derivata prima in funzione di alpha.

Ziosilvio 20-09-2007 12:23

Quote:

Originariamente inviato da pietro84 (Messaggio 18788660)
qualcuno sa dove posso trovare del materiale sui sistemi formali e sul teorema di incompletezza?

Per cosa ti serve?

Se è solo per cultura personale, puoi leggere "La prova di Gödel" di Nagel e Newman.

Se è per un esame, cerca "Introduzione alla logica matematica" di Elliott Mendelson.

BauAndrea 20-09-2007 12:26

grazie della spiegazione, durante lo svolgimento mi ero perso che quando k usciva da quei valori il discriminante di x diventava negativo. grazie ancora

pietro84 20-09-2007 13:06

Quote:

Originariamente inviato da Ziosilvio (Messaggio 18788790)
Per cosa ti serve?

Se è solo per cultura personale, puoi leggere "La prova di Gödel" di Nagel e Newman.

Se è per un esame, cerca "Introduzione alla logica matematica" di Elliott Mendelson.


grazie ora cerco, è per un esame.

poffarbacco 20-09-2007 13:52

Quote:

Originariamente inviato da Ziosilvio (Messaggio 18788758)
Essendoci di mezzo un logaritmo di x, la funzione è definita solo per x>0.
Se vuoi estenderla per continuità nel modo proposto, devi dimostrare che f(x) converge a 0 per x che tende a 0 da destra; e questo dovrebbe essere un classico.

Purtroppo sono un pò arrugginito, mi spiegheresti meglio questo punto?

Ziosilvio 20-09-2007 14:29

Quote:

Originariamente inviato da poffarbacco (Messaggio 18790628)
Purtroppo sono un pò arrugginito, mi spiegheresti meglio questo punto?

Questa ruggine su una materia di esame, però, non va bene...

Allora: abbiamo detto che, per x>0, è definita



Bisogna dimostrare che



Ora: di tre addendi, due tendono a zero per x-->0+, quindi occorre e basta dimostrare che



E questo è un classico, che si dimostra usando bene i limiti notevoli e le proprietà del logaritmo.

poffarbacco 20-09-2007 14:32

Quote:

Originariamente inviato da Ziosilvio (Messaggio 18791443)
Questa ruggine su una materia di esame, però, non va bene...

Allora: abbiamo detto che, per x>0, è definita



Bisogna dimostrare che



Ora: di tre addendi, due tendono a zero per x-->0+, quindi occorre e basta dimostrare che



E questo è un classico, che si dimostra usando bene i limiti notevoli e le proprietà del logaritmo.

Sinceramente non devo farlo io l'esame:D , è per la mia ragazza.
Scusa se non mi sono "applicato" e ho approfittato delle tue conoscenze, ma non avevo proprio tempo di rispolverare vecchie nozioni...grazie mille!

Hell-VoyAgeR 21-09-2007 13:15

una piccola comunicazione di servizio....

in questi giorni il servizio mimetex potrebbe subire delle interruzioni... sto montando il nuovo server...

come nuovo servizio ci sara' una piccola area del sito per la composizione della formula in tex, traduzione della url con i codici adatti per i.e. e firefox da inserire poi nel forum con un semplice copia e incolla! :)

ciaps

Lucrezio 21-09-2007 15:36

Quote:

Originariamente inviato da Hell-VoyAgeR (Messaggio 18805931)
una piccola comunicazione di servizio....

in questi giorni il servizio mimetex potrebbe subire delle interruzioni... sto montando il nuovo server...

come nuovo servizio ci sara' una piccola area del sito per la composizione della formula in tex, traduzione della url con i codici adatti per i.e. e firefox da inserire poi nel forum con un semplice copia e incolla! :)

ciaps



Grande :D
Grazie mille a nome di tutto il forum ;)

CioKKoBaMBuZzo 21-09-2007 15:38

scusate un attimo ma per dimostrare che non esiste l'estremo superiore di un insieme X (sottoinsieme di Q+) formato dagli x tali che x^2<2, non basta dimostrare che non esiste nessun numero razionale y tale che y^2=2?

Ziosilvio 21-09-2007 16:16

Quote:

Originariamente inviato da CioKKoBaMBuZzo (Messaggio 18808759)
per dimostrare che non esiste l'estremo superiore di un insieme X (sottoinsieme di Q+) formato dagli x tali che x^2<2, non basta dimostrare che non esiste nessun numero razionale y tale che y^2=2?

Non mi è chiaro il contesto.
In effetti, tale estremo superiore esiste (e non è un massimo) in IR, ma non appartiene (come giustamente fai osservare) ai razionali.
Per cui, se vedi X solo come sottoinsieme di Q, te la cavi dicendo "se esistesse, sarebbe un numero razionale il cui quadrato vale 2", ma se vedi X come sottoinsieme di IR, non te la cavi così a buon mercato.
Non è che l'esercizio chiedeva di dimostrare che non esiste il massimo di X?


Tutti gli orari sono GMT +1. Ora sono le: 09:30.

Powered by vBulletin® Version 3.6.4
Copyright ©2000 - 2024, Jelsoft Enterprises Ltd.
Hardware Upgrade S.r.l.